LSAT Logical Reasoning : Determining which answer most weakens the argument

Study concepts, example questions & explanations for LSAT Logical Reasoning

varsity tutors app store varsity tutors android store

Example Questions

1 2 4 Next →

Example Question #31 : Weaken/Undermine

The horrors of World War I have been blamed on many of the generals who commanded the armies in the trenches that became so prevalent in that conflict. While it is often said that generals fight the last war, the leaders arguing for bayonet charges into the teeth of a machine gun nest seemed to be unable to see the current war. Trench warfare was unable to be won through Napoleonic tactics, which was sadly proven beyond a shadow of a doubt in the Great War.

Which of the following statements would most weaken the argument presented in the statement?

Possible Answers:

World War I saw a number of generals take command of the armies with little success.

The military academy education of World War I generals was largely based on Napoleonic tactics.

Many generals in World War I began arguing against charges into machine gun nests.

The use of trenches radically transformed military strategy and tactics.

Training for soldiers at the beginning of World War I heavily featured bayonet training.

Correct answer:

Many generals in World War I began arguing against charges into machine gun nests.

Explanation:

The passage argues that the unsuccessful tactics of World War I, including bayonet charges into machine gun nests, were largely due to the generals in the war not understanding the problems with these tactics. If many generals were in fact arguing the utility of such tactics, then the argument would be seriously weakened.

Example Question #31 : Weaken/Undermine

Coffee allows people to stay awake, when they would otherwise have had to go to sleep, because of its caffeine content. According to several recent studies, people who drink coffee actually go to sleep earlier than people who drink no coffee. Therefore, coffee actually makes people go to sleep.

The strongest criticism of the argument is that it:

Possible Answers:

Concludes that people who do not drink coffee are not drinking other beverages which provide them with a source of caffeine.

Fails to consider other factors which might make someone drink coffee yet also go to sleep early.

Claims something is true without scientific evidence of the proof of it’s factual truth.

Cites several studies that are inconsistent with its conclusion.

Assumes caffeine is present in all coffee.

Correct answer:

Fails to consider other factors which might make someone drink coffee yet also go to sleep early.

Explanation:

There are facts in this problem which are intended to throw you off. The caffeine in coffee in this case does not matter. The studies only concluded that people who drink coffee go to sleep earlier than people who do not drink coffee. The problem does not state why, it could be that people who drink coffee are avoiding the higher caffeine content of energy drinks that everyone else drinks or it could be something else. That coffee makes people go to sleep is never stated as a fact. A causal relationship between coffee and sleep was never established.

Example Question #31 : Determining Which Answer Most Weakens The Argument

Students have been shown to become tired shortly after lunch, losing productivity in their afternoon classes and falling behind in these subjects compared to those they have first thing in the morning. Adding physical activities and break periods after lunch should make students perform much better in school.

Which of the following statements would most undermine the conclusion of the above passage?

Possible Answers:

Student disciplinary problems occur most often in classes directly after lunch.

Students often rate the subjects they take after lunchtime as their least favorite in surveys.

The success of students is highly related to the specific subject being taught.

Teachers express the most frustration with students in afternoon classes.

Academic success is caused by a number of interrelated, separate factors.

Correct answer:

The success of students is highly related to the specific subject being taught.

Explanation:

The passage states that students do not do well in their classes after lunch, and that changing these subjects could lead to more academic success. The argument hinges, therefore, on the effect of time of day on student achievement. A statement that would most undermine the argument must be the one that states academic success had no correlation with the time of day.

Example Question #32 : Determining Which Answer Most Weakens The Argument

Motorcycles have repeatedly been proven to be much more dangerous for their drivers than cars. While never as popular a vehicle on the road, motorcycles have a disproportionate number of casualties and fatalities as compared to automobiles. Traffic safety would be greatly improved if motorcycles were banned.

Which of the following statements most undermines the argument in the passage?

Possible Answers:

The traffic safety records of automobiles are not as good as other forms of transportation.

Motorcycle accidents do not always result in casualties or fatalities.

The safety record of motorcycles is much worse than all other forms of transportation.

Well-trained motorcyclists very rarely have accidents resulting in casualties or fatalities.

Motorcycles have always had a mixed safety record which has paled in comparison to automobiles.

Correct answer:

Well-trained motorcyclists very rarely have accidents resulting in casualties or fatalities.

Explanation:

The argument in the passage is that motorcycles are much more unsafe than automobiles, which means that they should be banned. A statement that shows well-trained motorcyclists do not have as bad a traffic safety record would directly undermine the statement that they are so dangerous they should be banned.

Example Question #32 : Weaken/Undermine

The vast majority of cancer drugs have proven to be ineffective. It makes sense, therefore, to turn to natural remedies as a possible solution. In a recent study, 90% of individuals without cancer had normal to high levels of iron in their blood, while only 20% of individuals with cancer had similarly high levels. Clearly, then, supplementing one's diet with iron can help prevent cancer.

Which of the following most weakens the argument outlined above?

Possible Answers:

The study was not conducted by doctors certified in the treatment of cancer.

Some of the cancer patients exhibited exceptionally high levels of iron in their blood.

The study was conducted with a small sample population. 

Almost all cancer treatments reduce a patient's iron levels. 

Iron can also reduce incidences of heart disease, stroke, and diabetes. 

Correct answer:

Almost all cancer treatments reduce a patient's iron levels. 

Explanation:

The correct answer disputes the causation implied by the arguments conclusion. Perhaps lower iron levels came after a patient developed cancer. In that case, it is not reasonable to infer that increasing one's intake of iron would prevent the development of cancer.

1 2 4 Next →

Tired of practice problems?

Try live online LSAT prep today.

1-on-1 Tutoring
Live Online Class
1-on-1 + Class
Learning Tools by Varsity Tutors